zuckermaus22
Thanks Received: 0
Vinny Gambini
Vinny Gambini
 
Posts: 3
Joined: December 28th, 2009
 
 
 

PT 37, S3, G4 - Swim Team Game

by zuckermaus22 Mon Jan 25, 2010 2:08 pm

I don't even know how to approach this one, though it seems really easy. Any advice?
 
aileenann
Thanks Received: 227
Atticus Finch
Atticus Finch
 
Posts: 300
Joined: March 10th, 2009
 
This post thanked 3 times.
 
 

Re: PT 37, S3, G4 - Swim Team Game

by aileenann Tue Jan 26, 2010 12:03 pm

Hello!

I can certainly understand the confusion - this is something of a funky game because of the element that the swimmers effectively repeat.

I'm attaching a diagram of what I did. The key is to make it visual. I think once you do that, you realize that mostly what you need to do is worry about assigning the first 5 and all else will follow from that. The only caveat is that of course you need to watch the 5/6 spots to see if you break any rules there that you don't break in the ordering of the first 5.

Take a look at this, and see if you might even make some additions to the diagram I've posted. Then take a crack at some of the problems. Sometimes if you're confused it helps to do the problems without time pressure to give yourself the comfort to really take time with a problem to think out all the nuances.

Please let me know if you still have questions after working through the diagram and trying out a problem or two!
Attachments
PT37, G4 -Swimmers-ManhattanLSAT.pdf
(50.82 KiB) Downloaded 920 times
 
ElianaG559
Thanks Received: 0
Vinny Gambini
Vinny Gambini
 
Posts: 1
Joined: December 03rd, 2018
 
 
 

Re: Diagram

by ElianaG559 Sun Jan 13, 2019 9:32 am

Regarding Question 23, I understand based on the diagram why J cannot occupy the 2/7 space, but I’m nit sure how option A (lap 1) also isn’t a valid answer given that O and M are before J in the diagram.
User avatar
 
ohthatpatrick
Thanks Received: 3807
Atticus Finch
Atticus Finch
 
Posts: 4661
Joined: April 01st, 2011
 
 
 

Re: Diagram

by ohthatpatrick Mon Feb 11, 2019 5:20 pm

Check out the thread for Q23

https://www.manhattanprep.com/lsat/foru ... t4554.html

The confusion you're having is stemming from the fact that every one goes twice.
The rules you're seeing on the diagram involve "at least one".

M's 1st is before O's 1st.
And at least one of J's is immediately after one of O's.
But that doesn't mean that both J's have to be after both O's.

here's a scenario that gets rid of (A)
J L M K O | J L M K O